011 Thoracic

   EMBED

Share

  • Rating

  • Date

    December 1969
  • Size

    170.1KB
  • Views

    905
  • Categories

Preview only show first 6 pages with water mark for full document please download

Transcript

Thoracic surgery 1. The bronchial circulation: A. Is the blood supply to the conducting airways. B. Drains into a peribronchial venous network that may expand considerably with conditions such as bronchiectasis and chronic obstructive pulmonary disease. C. Is an especially important consideration in pulmonary transplantation. D. All of the above. Answer: D DISCUSSION: The bronchial circulation is the primary blood supply for the conducting airways, pulmonary vessels, lymphoid tissue, and squamous cell carcinomas. In conditions such as mitral stenosis, bronchiectasis, or chronic obstructive pulmonary disease, the rich peribronchial venous network that drains the bronchial circulation may expand considerably, creating significant left-to-right shunts. Whenever the pulmonary artery circulation is obstructed, there is a tendency for bronchial circulation to increase; thus, the bronchial circulation is an important consideration during lung transplantation as well as in the surgical treatment of cyanotic congenital heart disease and chronic pulmonary embolism. 2. Clearance of mucus produced in the tracheobronchial tree in chronic bronchitis secondary to smoking may: A. Be hampered by the fact that the amount of mucus is increased by the number of mucus-producing cells at the expense of ciliated cells. B. Be slowed if patients have decreased lung volume and are therefore unable to generate a vigorous cough that would cause an inflammatory process. C. Cause a decrease in diffusion capacity and associated hypoxemia. D. All of the above. Answer: A DISCUSSION: Chronic bronchitis may have an acute component, and in these patients therapy with antibiotics and bronchodilators may improve the flow rate as measured by pulmonary function tests within 3 or 4 days of the cessation of smoking and treatment of the acute condition. However, the chronic bronchitic will continue to produce large amounts of mucus, most evident in the morning, even after the acute process has been resolved. Clearance of these secretions is hampered by the inability to cough, perhaps secondary to the pain of thoracotomy or abdominal surgery or by a decrease in the number of ciliary cells that help move mucus up the tracheobronchial tree. This causes plugging of small airways and atelectasis, which may progress to pneumonia. For this reason, cessation of smoking for 3 to 5 days before surgery is very beneficial in preventing pulmonary complications during the postoperative period. 3. The pulmonary circulation: A. Is the only vascular system in which the veins do not have the same course as the arteries. B. Has a direct connection of vein to adjacent lung tissue by connective tissue fibers, making the diameter of the tissue fibers dependent upon lung volume. C. Supplies the metabolic needs of the alveoli. D. All of the above. Answer: C DISCUSSION: Pulmonary artery circulation transports oxygenated blood to the alveoli level where gas exchange occurs, and it is here that the matching of ventilation and perfusion is so important during the postoperative period. The loss of lung volume that generally occurs after all surgical procedures does not return to baseline for 5 to 7 days and may play an important role in the ventilation-perfusion ratio. Improving or returning lung volume to normal is performed by manipulating functional residual capacity (FRC) and preventing atelectasis, which in turn maintains circulation to the alveolus and optimizes the ventilation-perfusion ratio. 4. Which of the following screening tests are important for preoperative evaluation of pulmonary function? A. History and physical examination. B. Room air arterial blood gases. C. Chest film. D. Vital capacity and forced expiratory volume in 1 second (FEV 1). E. Cardiopulmonary exercise testing. Answer: ABCDE Asir Surgery MCQs Bank. © 1422H-2002- first impression © This project was raised after an idia by Dr. Gharama Al-Shehri (consultant surgeon). Developed and typed by Dr. Ghazi Al-Shumrani (intern). 97 Thoracic surgery DISCUSSION: The most important clues to impairment of respiratory function are found in the history and physical examination. A negative history and physical examination in combination with a relatively normal room air arterial blood gas and normal chest film are sufficient to screen patients to support the clinical impression that there is minimal pulmonary disease. Patients with symptoms, positive physical findings, and/or abnormalities in the arterial blood gases or chest film can be screened most effectively with an additional evaluation of the vital capacity and FEV 1. More elaborate tests such as cardiopulmonary exercise testing are reserved for patients with obvious and marked impairment of pulmonary function who are being evaluated for the feasibility of surgical intervention. 5. Carbon monoxide diffusion capacity (DLCO) has been shown to correlate with: A. The thickness of the alveolar lining membrane. B. The permeability of the erythrocyte to carbon dioxide. C. Pulmonary emboli. D. Total alveolar-capillary capacity. Answer: ABCD DISCUSSION: The single-breath DLCO is a screening test that has been shown to be decreased in all of the above examples. It is an estimate of the total capacity of the functional alveolar microarchitecture and has been demonstrated to be an independent measure of physiologic capability apart from the FEV 1 and forced ventilatory capacity. 6. The closing volume is: A. The volume remaining in the lung at the end of expiration below which alveolar collapse begins to occur, resulting in physiologic shunting. B. Higher in young persons. C. Not changed during surgery. D. Relative to the oxygen content of mixed venous blood. Answer: AC DISCUSSION: The closing volume is conceptually the remaining lung volume at the end of expiration below which alveolar collapse begins to occur, causing intrapulmonary right-to-left shunting and thus desaturation of blood in the left atrium. In a normal young person this closing volume is well below the functional residual capacity (FRC); thus, such physiologic shunting does not occur until there is a decrease in the elastic properties of the lung. Although FRC gradually increases with age, so does the effective closing volume. Eventually some alveoli are being underventilated (at endexpiration), allowing physiologic right-to-left shunting to occur. Closing volume is unchanged, but FRC decreases during surgery (i.e., shunting occurs). Closing volume has no direct relationship to the oxygen content of the mixed venous blood. 7. The effect of high positive end-expiratory pressures (PEEP) on cardiac output is: A. None. B. Increased cardiac output. C. Decreased cardiac output because of increased afterload to the left ventricle. D. Decreased cardiac output because of decreased effective preload to the left ventricle. Answer: D DISCUSSION: Higher levels of PEEP can be associated with decreases in cardiac output as a consequence of an effective decrease in the preload to the left ventricle owing to impaired left ventricular filling. 8. Weaning patients from maximum ventilator support usually involves: A. Weaning PEEP first, tidal volume second, and the fraction of inspired oxygen (FIO 2) third. B. Weaning FIO 2 first, ventilator rate second, and PEEP third. C. Weaning FIO2 first, PEEP second, and tidal volume third. D. Weaning FIO 2 first, PEEP second, and ventilator rate third. Answer: D Asir Surgery MCQs Bank. © 1422H-2002- first impression © This project was raised after an idia by Dr. Gharama Al-Shehri (consultant surgeon). Developed and typed by Dr. Ghazi Al-Shumrani (intern). 98 Thoracic surgery DISCUSSION: When the inspired oxygen concentration is greater than 60% for more than 24 to 28 hours, the risk of oxygen toxicity increases. PEEP is usually weaned to physiologic levels (i.e., 5 to 7 cm. H 2O) before weaning either rate or tidal volumes. Generally, the optimal tidal volume to achieve alveolar recruitment is selected and usually is not decreased unless peak airway pressures increase. If decreases in ventilatory rate are not tolerated, airway pressure support can be added. 9. Which of the following statements about bronchoscopy is false? A. The morbidity and mortality are approximately 0.2% and 0.08%, respectively. B. The most common complications of bronchoscopy are related to premedication of patients. C. Adjunctive cancer therapy such as laser treatment and brachytherapy may be administered via this route. D. A chronic cough and unilateral wheezing are accepted indications for bronchoscopy. E. Early postoperative bronchoscopy for atelectasis is contraindicated following pulmonary resection. Answer: E DISCUSSION: The stated complication rates are true and reported from a comprehensive review of over 24,000 patients. Although the most common complications are related to the premedication, significant hemorrhage, pneumothorax, bronchospasm, and dysrhythmias have been reported. In addition to laser and brachytherapy, phototherapy and immunotherapy have been given by bronchoscopy. Unilateral wheezing may represent a bronchial foreign body, and a chronic cough could signify myriad pulmonary disorders. Accordingly, both are amenable to diagnostic bronchoscopy. Most thoracic surgeons favor early bronchoscopy for lobar atelectasis following pulmonary surgery. 10. Flexible bronchoscopy is preferred over rigid bronchoscopy for all of the following except: A. Patients with cervical spine injuries requiring intubation. B. The evaluation of a smoke inhalation injury. C. Transcarinal needle aspiration of an enlarged subcarinal lymph node. D. The removal of a bronchus intermedius foreign body from an infant. E. A cost-effective evaluation of mild hemoptysis. Answer: D DISCUSSION: Neither patients with significant cervical spine disease or injuries nor those with large aortic arch aneurysms should undergo rigid bronchoscopy, given the greater risk of complications. Even if severe, smoke inhalation injury can be assessed adequately by flexible bronchoscopy. Transbronchial needle aspiration of lesions that on computed tomography (CT) look suspicious is safe and quite easily performed with fluoroscopic guidance. Hemoptysis as a presenting symptom should be evaluated by flexible bronchoscopy. In contrast, for massive hemoptysis an airway should be secured for ventilation with a rigid bronchoscope. Similarly, an airway needs to be maintained while removing endobronchial foreign bodies from infants or children. Since adequate port sites for instrumentation are also needed, the rigid bronchoscope is preferred in this setting. 11. Which of the following approaches is/are currently acceptable for the management of spontaneous pneumothorax? A. Chest tube replacement alone for the patient with a first episode. B. Operation on presentation for any patient with a first episode. C. Video-assisted thoracic surgery (VATS) bleb excision and pleurodesis for recurrent pneumothorax on the same side. D. Thoracotomy with bleb excision and pleurodesis for unilateral recurrent pneumothorax. E. Operation after a first episode in an airline pilot. Answer: ACDE DISCUSSION: Primary spontaneous pneumothorax typically occurs in young patients with congenital blebs at the apices of the lungs. Rupture of these blebs causes pneumothorax, which recurs in about 30% of patients. Standard care on initial presentation is chest tube placement alone. Operation traditionally has been performed during the initial episode only if there is prolonged air leakage (longer than 7 days); in patients with bilateral pneumothorax; those who live in an area where prompt medical care, if needed, is not available; and those frequently exposed to extremes of pressure (e.g., airline pilots). Both VATS and thoracotomy approaches to excision of blebs and pleurodesis have been shown to be effective. The availability of the apparently safer VATS approach has led some to favor earlier operation. Asir Surgery MCQs Bank. © 1422H-2002- first impression © This project was raised after an idia by Dr. Gharama Al-Shehri (consultant surgeon). Developed and typed by Dr. Ghazi Al-Shumrani (intern). 99 Thoracic surgery 12. For which patient(s) with a pulmonary infiltrate of uncertain cause would you favor VATS over open wedge excision? A. An AIDS patient with a diffuse infiltrate who is ambulatory but requires supplemental oxygen. Bronchoalveolar lavage is negative. B. A 64-year-old previously healthy man with increasing shortness of breath, a diffuse infiltrate, and restrictive lung disease as shown by pulmonary function studies. C. A 74-year-old diabetic woman with a rapidly progressing process throughout the right lung who is ventilator- and pressor-dependent. D. A 44-year-old man with fever, left-sided infiltrate, and shortness of breath. E. A 79-year-old man on a ventilator for right lower and middle lobe pneumonia which has been culture negative. Answer: ABD DISCUSSION: Lung biopsy by VATS or minithoracotomy is often indicated in the work-up of a pulmonary infiltrate that has not been successfully diagnosed by less invasive studies. This procedure probably is not indicated for cancer patients with acute pneumonitis, as broad-spectrum antibiotics frequently are successful treatments. For those who do require the procedure, the choice between VATS and thoracotomy is determined by the severity of illness. In those who are critically ill and ventilator dependent, changing the tube to an endobronchial tube for thoracoscopy may be risky, and in these cases an anterior thoracotomy with single-lumen ventilation is indicated. These patients, who are frequently heavily sedated and are likely to remain so for some time postoperatively, are unlikely to benefit from the greatest advantage of VATS, the reduction of postoperative pain. It is, then, the ambulatory patient with a chronic interstitial process who benefits the most from the VATS approach. 13. Which of the following statements about the cause and prevention of postintubation tracheal stenosis are correct? A. Postintubation airway stenosis can largely be avoided by providing assisted ventilation via endotracheal tube rather than tracheostomy tube. B. Postintubation tracheal stenosis at the cuff level results, more or less equally, from low blood pressure, advanced age, steroids, high intracuff pressure, sensitivity to tube materials, gas sterilization elution products, and systemic disease. C. In women and smaller men large endotracheal tubes can produce lesions of the glottis and subglottis that can progress to stenosis. D. Stomal stenosis is due principally to cicatricial closure of large stomas resulting from removal of a disk or segment of tracheal wall during tracheostomy. E. A large-volume tracheostomy tube cuff such as that now used on most available tubes can become a high-pressure cuff if filled beyond its resting maximal volume. Answer: CE DISCUSSION: Use of an endotracheal tube, of course, avoids a stoma and related complications. Cuff lesions, however, are incurred from cuffs on endotracheal tubes, cricothyroidostomy tubes, and tracheostomy tubes. A cuff is the common factor. Endotracheal tubes, on the other hand, cause erosion at the level of the glottis and subglottis, in particular. Although many factors may play some role in the origin of airway stenosis, the principal factor is pressure necrosis of the mucosa, submucosa, and ultimately of the cartilage, with subsequent cicatrization. Large endotracheal tubes do, indeed, cause necrosis and airway injury at narrow areas in the upper airway, at glottic and cricoid levels. Such injuries lead to posterior commissure stenosis, arytenoid fixation, vocal cord erosion and granulomas, anterior commissure stricture, and subglottic circumferential stenosis. While excision of a large amount of tracheal wall can lead inevitably to healing by contraction with narrowing of the tracheal circumference, the most usual cause of stomal stenosis is erosion of the stoma by pressure from the tracheostomy tube. This in turn may relate to leverage by equipment. Additional factors appear to be subsidiary. A large volume, low pressure tracheostomy tube cuff, such as those currently available, if properly constructed, will seal the trachea before it is necessary to stretch the cuff by adding an increasing volume of air. If the cuff is stretched beyond that resting volume, which usually occludes the normal trachea, high pressures will develop because the plastic material from which all of these cuffs are now made is not very extensible. Therefore, the pressure-volume curve rises sharply once the limit of unstretched volume is passed. A low-pressure cuff then becomes a high-pressure cuff. 14. Which of the following statements about the treatment of postintubation airway stenosis are correct? A. Emergency management of airway obstruction due to stenosis at the level of a prior tracheal stoma is best accomplished by establishing a new tracheostomy in normal tracheal tissue just below the scar of the old stoma. B. Radial lasering and dilatation usually leads to permanent resolution of postintubation tracheal stenosis. C. Splinting of a cervical trachea with a silicone T-tube for 6 to 8 months generally leads to permanent resolution of stricture. Asir Surgery MCQs Bank. © 1422H-2002- first impression © This project was raised after an idia by Dr. Gharama Al-Shehri (consultant surgeon). Developed and typed by Dr. Ghazi Al-Shumrani (intern). 100 Thoracic surgery D. Postintubation tracheal stenosis that extends into the subglottic larynx is treated by resection of a cylindrical sleeve of stenotic airway and end-to-end reconstruction. E. Acquired tracheoesophageal fistula due to intubation injury is corrected by surgical closure of the fistula concurrent with resection and reconstruction of the damaged trachea. Answer: E DISCUSSION: Emergency management of postintubation tracheal stenosis is accomplished by dilatation under general anesthesia using rigid bronchoscopes and dilators. Tracheostomy is employed only when the patient requires a prolonged or a permanent airway before or instead of resection and reconstruction. If a new tracheostomy is needed it is preferable to place it through the area of existing stenosis—in this case the site of prior tracheostomy—rather than to injure normal trachea that will be needed for resection and reconstruction. If the stenosis is below the sternal notch, a long tracheostomy tube is inserted at the usual position (second and third rings) but extends past the now dilated stenotic lesion. Lasering almost never results in a permanently satisfactory airway; the exception is the very limited lesion described as a thin, weblike stenosis. Such lesions are rare. Unfortunately, laser is widely used and often compounds existing damage by concurrent placement of a tracheostomy tube below the lesion in the normal trachea. T-tubes are very useful for temporizing when repair is not possible or must be delayed. It almost never leads to resolution of a stricture unless it is an extremely limited one. Soon after removal of the T-tube the stricture reasserts itself. If the lesion involves the subglottic larynx, complex repair is required to preserve the recurrent laryngeal nerve's anatomy and function. The posterior cricoid plate is salvaged and resurfaced with a flap of membranous tracheal wall; the anterior subglottic larynx is reconstructed with a “prow” of distal tracheal cartilage and mucosa. A tracheoesophageal fistula is managed (after weaning from a respirator) by layered closure of the esophagus, interposition of a flap of well-vascularized tissue (such as a pedicled strap muscle), and resection and reconstruction of the damaged trachea. Since the fistula results from the pressure of a cuff, often against an esophageal feeding tube, there is circumferential damage to the trachea at the level of the fistula. Resection and reconstruction are therefore necessary, in addition to closure of the fistula, for successful treatment of this complex lesion. 15. Which of the following statements are true? A. Pyogenic lung abscess occurs most frequently in the lower lobe of the left lung. B. Anaerobic bacteria are commonly present in pyogenic lung abscess. C. Operation is usually required to eradicate a pyogenic lung abscess. D. Penicillin is the treatment of choice for lung abscess. Answer: BD DISCUSSION: Pyogenic lung abscess is the result of aspiration of debris from the oropharynx. Since most patients are unconscious and supine when this occurs, the aspirated material usually finds its way into the most dependent bronchi. These are the superior division of the right lower lobe and the posterior segment of the right upper lobe. The organisms most commonly responsible for pyogenic lung abscess are the same anaerobic bacteria found in the mouths of patients with poor oral hygiene. In addition to anaerobic organisms alpha- and beta-hemolytic streptococci, staphylococci, nonhemolytic streptococci, and Escherichia coli may be present. Gram-negative rods and staphylococci are particularly common in hospital-acquired infections. These organisms are almost always penicillin sensitive. Surgical therapy is rarely necessary to eradicate a pyogenic lung abscess. Penicillin, alone or in combination with metronidazole, is the drug of choice. Metronidazole alone probably lacks sufficient activity against anaerobic and microaerophilic streptococci. Clindamycin is also effective against most anaerobic bacteria present in pyogenic lung abscesses. 16. Amphotericin B is effective for the following lung infections: A. Histoplasmosis. B. North American blastomycosis. C. Aspergillosis. D. Mucormycosis. E. Sporotrichosis. Answer: ABD DISCUSSION: Amphotericin B is effective against most fungal infections, including histoplasmosis and North American blastomycosis. Aspergillosis is caused by the fungus Aspergillus fumigatus, an organism that is resistant to treatment with iodides, nystatin, hydroxystilbamidine, and amphotericin B. The treatment for this fungal infection is a surgical procedure, if the patient's condition permits. Surgical excision and amphotericin B usually are necessary to treat mucormycosis. Asir Surgery MCQs Bank. © 1422H-2002- first impression © This project was raised after an idia by Dr. Gharama Al-Shehri (consultant surgeon). Developed and typed by Dr. Ghazi Al-Shumrani (intern). 101 Thoracic surgery Itraconazole is the drug of choice for sporotrichosis. Itraconazole, ketoconazole, and fluconazole should be considered as primary or secondary drugs when treating systemic fungal infections. 17. The following statements are true. A. A distinguishing roentgenographic appearance of lung abscess, the air-fluid level can be seen only on roentgenograms obtained in the upright or lateral decubitus position. B. The fungus ball characteristic of aspergillosis can be seen roentgenographically in either the upright or recumbent position. C. Actinomycosis and nocardiosis are both fungal diseases of the lung that respond to treatment with the newer azole antifungal agents. D. The commonest fungal lung infection in the United States is due to Histoplasma capsulatum. Answer: ABD DISCUSSION: An air-fluid level distinguishes a lung abscess. While this can be seen easily in an upright or lateral decubitus roentgenogram, it cannot be seen when the patient is recumbent. The fungus ball characteristic of aspergillosis is identified by its crescent-shaped shadow on a roentgenogram. When the patient changes from an upright to recumbent position, the fungus ball may also change position in the cavity in the lung. Both actinomycosis and nocardiosis are bacterial infections and do not respond to antimycotic treatment. Actinomyces israelli is treated with penicillin and Nocardia asteroides is sensitive to trimethoprim-sulfamethoxazole. The most common fungal infection in North America is histoplasmosis. More than 30 million people have been infected, most of whom are asymptomatic. 18. Pneumocystis pneumonia is an opportunistic infection caused by Pneumocystis carinii. Which of the following statements are true? A. P. carinii is a fungus. B. Pneumocystis pneumonia is the most common opportunistic infection in patients with AIDS. C. The diagnosis of Pneumocystis pneumonia depends on the demonstration of P. carinii organisms in lung tissue. D. There is no effective treatment for Pneumocystis pneumonia. Answer: BC DISCUSSION: P. carinii is a protozoan that stains with silver methenamine and resembles a fungus. It responds to antiprotozoal drugs. Pneumocystis pneumonia occurs in 80% of AIDS patients. The diagnosis is made by demonstrating the organisms in lung tissue by transbronchoscopic lung or brush biopsy, percutaneous needle biopsy, or open lung biopsy. Both trimethoprin-sulfamethoxazole and pentamidine isethionate are effective against P. carinii. 19. Which of the following statements are true? A. The pleural space does not extend into the neck. B. Positive intrapleural pressures as high as 40 cm. H 2O and negative pressures as low as -40 cm. H 2O are possible. C. The pleural cavities cannot absorb more than 500 ml. of fluid per day. D. All pleural effusions are of clinical significance and should be investigated. Answer: BD DISCUSSION: The pleural spaces extend into the neck as well as retrosternally and into the costophrenic sinuses. This should be kept in mind when performing procedures such as subclavian and jugular puncture, to avoid pneumothorax. Markedly elevated intrapleural pressures are obtained with the Valsalva maneuver, and extreme negative pressures can be produced with forced inspiratory effort against a closed glottis. Because of the many microvilli present on the mesothelial cells that line the pleural cavity, a liter or more of fluid is easily secreted or absorbed within a 24-hour period. Most pleural effusions are caused by infection, tumor, or congestive heart failure and should be investigated to determine the proper course of management. 20. Which of the following statements are true? A. Chylothorax, or chyle in the pleural cavity, usually is not a serious condition. B. Chyle is easily identified by its milky appearance, which looks like no other kind of pleural effusion. C. The commonest causes of chylothorax are trauma and tumor. D. The thoracic duct can be ligated with impunity. Asir Surgery MCQs Bank. © 1422H-2002- first impression © This project was raised after an idia by Dr. Gharama Al-Shehri (consultant surgeon). Developed and typed by Dr. Ghazi Al-Shumrani (intern). 102 Thoracic surgery Answer: CD DISCUSSION: Chylothorax is most often the result of trauma; however, spontaneous chylothorax is also a manifestation of tumor and should be investigated to identify occult malignancies. Chyle in the thorax is characteristically milky white but can be mistaken for the pseudochylothorax of rheumatoid disease or tuberculosis. If necessary, a diagnosis can be confirmed by lymphangiography. This also facilitates ligation of the thoracic duct, should this become necessary to control the loss of chyle. 21. Which of these statements about pleural tumors is/are true? A. The commonest type of pleural tumor is primary pleural mesothelioma. B. Exposure to asbestos dust is causally related to the development of malignant mesothelioma. C. Localized benign mesotheliomas are asymptomatic. D. Complete pleurectomy for malignant mesothelioma usually results in cure. Answer: B DISCUSSION: Pleural involvement by metastatic disease is much more common than primary pleural tumors. Patients with localized benign pleural mesotheliomas may have symptoms of arthralgia, clubbing of the fingers, or fever, which usually disappear after excisional surgery. The evidence relating industrial exposure to asbestosis and malignant pleural mesothelioma is quite strong. Excisional surgery for malignant mesothelioma is usually only palliative. Most patients succumb within 1 to 2 years of the diagnosis, regardless of the kind of treatment they receive. 22. Which of the following correctly describe a patient with spontaneous pneumothorax? A. The patient is almost always elderly and debilitated. B. An unsuspected primary or metastatic lung tumor may be present. C. The administration of supplemental oxygen is of little benefit to the patient. D. The patient should always be treated with an intercostal tube and closed pleural drainage. E. Video-assisted thoracic surgery (VATS) should be considered for persistent air leak in patients with secondary spontaneous pneumothorax. Answer: BE DISCUSSION: A patient with spontaneous pneumothorax may be old and debilitated, but the typical patient is an otherwise healthy young adult, usually one who smokes. An incidental, unsuspected lung cancer is discovered on rare occasions when operation is performed to control a persistent air leak. Perhaps smoking is a common factor. Absorption of air from the pleural space can be facilitated by the administration of supplemental oxygen. Increasing the oxygen tension lowers the partial pressure of nitrogen (P N2) of the capillary blood and increases the partial pressure difference between the pleural space and the pulmonary capillary. If the pneumothorax results in less than 20% collapse of the lung an asymptomatic patient can be safely observed; however, a larger or persistent pneumothorax is best treated with an intercostal tube thoracostomy. Patients with bullous emphysema may require stapling of bullae and pleurectomy, which can be done by open thoracotomy or thoracoscopically (VATS). 23. Which of the following statements about spontaneous pneumothorax (PSP) is/are correct? A. The risk of recurrence after resolution of the first episode of PSP or secondary spontaneous pneumothorax (SSP) is 35% to 45%. B. Patients with PSP are typically tall, thin, young adult males with a history of smoking. C. Secondary spontaneous pneumothorax is associated with family history in 10% of cases. D. For bleb resection and pleurodesis thoracoscopic thoracotomy and open thoracotomy provide similar cure rates for patients with primary spontaneous pneumothorax. E. Causes of secondary pneumothorax include trauma and iatrogenic needle puncture. Answer: ABD DISCUSSION: Patients with PSP are usually 20- to 40-year-old males with a common long-chested body habitus. The majority of PSP patients have a history of tobacco use and 10% have a family history of PSP. The majority of cases of SSP are due to advanced emphysema in a population of patients aged 50 to 70 years. Additional causes of SSP include tuberculosis, cystic fibrosis, P. carinii infection, lung cancer, and lung abscess. For patients with PSP bleb resection and Asir Surgery MCQs Bank. © 1422H-2002- first impression © This project was raised after an idia by Dr. Gharama Al-Shehri (consultant surgeon). Developed and typed by Dr. Ghazi Al-Shumrani (intern). 103 Thoracic surgery pleurodesis performed thoracoscopically provides cure rates similar to those of open thoracotomy. Because of the nature of underlying pulmonary diseases, open thoracotomy appears to provide better results for patients with SSP. 24. Which of the following are relative contraindications for surgical management of emphysema? A. Rapidly progressive dyspnea. B. Bullae occupying less than one third of a hemithorax on plain chest radiography. C. Elevated room air PCO 2. D. “Pink puffer” patients. E. FEV 1 less than 35% of predicted value. Answer: BCE DISCUSSION: Relative contraindications to operation for bullous emphysema include patients with carbon dioxide retention, FEV 1 less than 35% of predicted value, small bullae that occupy less than one third of a hemithorax, and “blue bloaters,” who are prone to the sequelae of chronic bronchitis. Patients who have primarily emphysema (“pink puffers”) and rapidly progressive dyspnea are usually good candidates for operation. 25. Which of the following treatments would be appropriate therapy for symptoms that persist on medical therapy and bronchiectasis involving, in order of decreasing severity, the left lower lobe, the right middle lobe, and the left upper lobe? A. Left pneumonectomy. B. Wedge resection of the left lower lobe. C. Left lower lobectomy. D. Simultaneous left lower lobectomy and right middle lobectomy. Answer: C DISCUSSION: Pneumonectomy is seldom indicated today for bronchiectasis. Anatomic resection of involved segments with either segmentectomy or lobectomy is preferred to nonanatomic wedge resection. Bilateral pulmonary resections should generally be done as staged procedures, the most symptomatic side being resected first. Then, the contralateral side is resected only if symptoms persist during a prolonged course of medical therapy. 26. Which of the following would not be acceptable sequences of preoperative studies in a patient being prepared for lingulectomy for bronchiectasis? A. CT alone. B. CT, bronchoscopy, bronchography. C. Bronchoscopy alone. D. Bronchoscopy, bronchography. Answer: C DISCUSSION: Bronchoscopy alone is generally not diagnostic for bronchiectasis. Thin-section, high-resolution CT can diagnose bronchiectasis and define the airway anatomy sufficiently for resection. Bronchography is performed less frequently today but can be very useful in diagnosing bronchiectasis and defining airway anatomy for pulmonary resection. 27. Which of the following statements about pulmonary mycobacterial infection is/are correct? A. Worldwide, tuberculosis no longer represents a significant public health problem. B. Mycobacterium tuberculosis is responsible for the majority of cases of pulmonary mycobacterial disease. C. Mycobacterium kansasii pulmonary infection almost always requires surgical treatment. D. Atypical mycobacteria are never primary pulmonary pathogens in humans. E. Mycobacterium avium-intracellulare is generally resistant to most antimycobacterial drugs in vitro. Answer: BE DISCUSSION: Tuberculosis remains the leading infectious killer in the world today. M. tuberculosis is responsible for the vast majority of pulmonary mycobacterial disease. M. kansasii infection responds to multiple drug chemotherapy and relatively infrequently requires surgical treatment. Atypical mycobacteria can be primary pulmonary pathogens in humans. M. avium-intracellulare is usually resistant in vitro to most antituberculosis drugs. Asir Surgery MCQs Bank. © 1422H-2002- first impression © This project was raised after an idia by Dr. Gharama Al-Shehri (consultant surgeon). Developed and typed by Dr. Ghazi Al-Shumrani (intern). 104 Thoracic surgery 28. Which of the following chemotherapeutic regimens are currently recommended for the treatment of pulmonary infection caused by M. tuberculosis? A. Isoniazid, rifampin, pyrazinamide, and streptomycin for 24 months. B. Isoniazid for 9 months with ethambutol for the first 3 months. C. Isoniazid and rifampin for 6 months with pyrazinamide added for the first two months. D. Isoniazid alternating with rifampin at 3-month intervals for 12 months. E. Isoniazid and rifampin for 9 months. Answer: CE DISCUSSION: Treatment of tuberculosis with a single drug leads to rapid emergence of drug-resistant organisms. Any treatment regimen that employs only one drug for a period of time encourages the development of drug-resistant tuberculosis. Currently, the American Thoracic Society recommends either (1) a 6-month regimen consisting of isoniazid, rifampin, and pyrazinamide for 2 months followed by isoniazid and rifampin for 4 months or, alternatively, (2) a 9-month course of isoniazid and rifampin. Prolonged courses of treatment beyond 9 to 12 months no longer are considered necessary. 29. Which of the following are appropriate indications for pulmonary resection for mycobacterial disease? A. Localized pulmonary disease caused by M. avium-intracellulare. B. Advanced lobar tuberculous pneumonia with massive hilar lymphadenopathy and bronchial obstruction in a young child. C. Localized pulmonary disease due to multiple drug–resistant M. tuberculosis. D. An asymptomatic tuberculous cavity greater than 12 cm. in diameter. E. Massive hemoptysis from a right upper lobe cavity occurring during an appropriate course of chemotherapy for pulmonary tuberculosis in a sputum-negative patient. Answer: ACE DISCUSSION: Modern antimycobacterial chemotherapy is very effective. Surgical treatment of pulmonary mycobacterial disease is rarely necessary; however, pulmonary disease caused by M. avium-intracellulare or multiple drug–resistant M. tuberculosis is not likely to respond to chemotherapy and should be resected if the disease is localized. Chemotherapy for tuberculosis is almost invariably curative in children, regardless of the extent of disease. The size of a tuberculous cavity is not an indication for resection. Massive hemoptysis from a cavitary lesion is life threatening and is an indication for pulmonary resection. 30. Which statements about squamous papillomatosis of the trachea is/are correct? A. It is the most common type of benign tracheal tumor in adults. B. It is the most common type of benign tracheal tumor in children. C. Most are treated with segmental tracheal resection. D. There is no risk of malignant degeneration. E. It is associated with a herpesvirus. Answer: A DISCUSSION: Squamous papillomatosis is the most common benign tracheal and bronchial tumor in adults. Up to 50% of untreated lesions may degenerate into squamous cell carcinoma. The lesion is associated with human papillomavirus types 6 and 11, and therefore, interferon therapy is under investigation. Most patients can be treated successfully by repeated bronchoscopic fulguration, laser ablation, or cryotherapy. 31. Which of the following statements about pulmonary hamartomas is/are true? A. Hamartomas are benign chondromas. B. Most are located in the conducting airways. C. Wedge resection is curative. D. A lobectomy is necessary to obtain draining hilar lymph nodes. E. Hemoptysis is common. Answer: C Asir Surgery MCQs Bank. © 1422H-2002- first impression © This project was raised after an idia by Dr. Gharama Al-Shehri (consultant surgeon). Developed and typed by Dr. Ghazi Al-Shumrani (intern). 105 Thoracic surgery DISCUSSION: Pulmonary hamartomas are benign masses consisting of cartilage, lymph tissue, fat, and epithelial elements. Eighty per cent are located in the lung periphery and are treated by a small wedge resection, usually with a thoracoscope. Most are asymptomatic, and there is no risk of malignant degeneration. 32. Which of the following statements about typical carcinoid tumors are true? A. They make up the majority of bronchial adenomas. B. They frequently have lymph node metastases. C. The carcinoid syndrome is observed in 33%. D. Overall survival at 5 years is 90%. E. Overall survival at 5 years is 50%. Answer: AD DISCUSSION: Eighty-five per cent of bronchial adenomas are carcinoid tumors. Typical carcinoid tumors have few mitotic figures and infrequent lymph node metastases (fewer than 10%). Only 10% to 15% of patients present with the carcinoid syndrome (flushing, wheezing, diarrhea). Survival after resection is more than 90% at 5 years but decreases to approximately 50% for atypical histology. 33. Which is/are true of adenoid cystic carcinoma? A. It is a common type of salivary gland tumor. B. Another name is cylindroma. C. Most patients are completely resected for cure. D. Different histological types have different prognoses. E. Tissue invasion is rare. Answer: ABCD DISCUSSION: Adenoid cystic carcinomas (cylindromas) are commonly observed salivary gland tumors that can occur in the conducting airways. The undifferentiated solid type is associated with distant metastases, of which the cribriform and tubular types are associated with perineural and submucosal invasion. Most patients (60%) can be resected for cure. 34. A solitary pulmonary nodule is discovered in an asymptomatic 55-year-old smoker with no evidence of extrathoracic dissemination. The most appropriate management would be to: A. Obtain serial chest films every 3 months to determine the growth potential of the nodule. B. Perform transthoracic needle aspiration (TTNA) before considering pulmonary resection to confirm malignancy. C. Conduct an extensive systematic evaluation to exclude the possibility that the nodule represents a metastatic lesion. D. Proceed with pulmonary resection after ascertaining that the patient would tolerate removal of the requisite amount of lung. E. Obtain baseline serum levels of carcinoembryonic antigen and p53. Answer: D DISCUSSION: A patient with a solitary pulmonary nodule—a single spherical lesion within the lung— represents an important and challenging diagnostic problem in thoracic oncology. A solitary pulmonary nodule is assumed to be primary lung cancer until proved otherwise; the differential diagnosis includes metastatic carcinoma, granuloma, and benign pulmonary tumors. In most cases, solitary pulmonary nodules should be resected after thorough investigation to establish that systemic dissemination has not already occurred. CT of the chest, liver, and adrenals is performed to confirm the location of the tumor, to evaluate the mediastinum, and to assess the abdomen for systemic disease. If there is no evidence of metastases on CT, the patient should undergo bronchoscopy, which may establish the histologic diagnosis and determine resectability if an endobronchial lesion exists. Pulmonary function studies are obtained preoperatively to assess the potential for pulmonary resection. A thorough review of systems is undertaken to rule out medical contraindications to thoracotomy. TTNA is not performed routinely and should be reserved for patients with marginal pulmonary function, for whom thoracotomy would be performed only after verification of a malignant histologic diagnosis. 35. After thoracotomy, pulmonary resection, and mediastinal lymph node dissection, a patient is determined to have a squamous cell carcinoma 2 cm. in diameter, located 1 cm. from the carina along the right mainstem bronchus. Three Asir Surgery MCQs Bank. © 1422H-2002- first impression © This project was raised after an idia by Dr. Gharama Al-Shehri (consultant surgeon). Developed and typed by Dr. Ghazi Al-Shumrani (intern). 106 Thoracic surgery peribronchial lymph nodes are positive for cancer, and all other lymph node stations are negative. The correct stage, according to the TNM system, is: A. T1N0M0 Stage I. B. T1N1M0 Stage II. C. T2N1M0 Stage II. D. T3N1M0 Stage IIIa. E. T2N3M0 Stage IIIb. Answer: C DISCUSSION: The TNM staging system for carcinoma of the lung provides a consistent, reproducible description of the anatomic extent of disease at the time of diagnosis. In the TNM system, T represents the primary tumor and numerical suffixes describe increasing size or involvement; N represents regional lymph nodes with suffixes to describe levels of involvement; and M designates the presence or absence of distant metastases. TUMOR (T) TX Occult carcinoma (malignant cells in sputum or bronchial washings but tumor not visualized by imaging studies or bronchoscopy) T1 Tumor 3 cm. or less in greatest diameter, surrounded by lung or visceral pleura, but not proximal to a lobar bronchus T2 Tumor larger than 3 cm. in diameter, or with involvement of main bronchus at least 2 cm. distal to carina, or with visceral pleural invasion, or with associated atelectasis or obstructive pneumonitis extending to the hilar region but not involving the entire lung T3 Tumor invading chest wall, diaphragm, mediastinal pleura, or parietal pericardium; or tumor in main bronchus within 2 cm. of, but not invading, carina; or atelectasis of obstructive pneumonitis of the entire lung T4 Tumor invading mediastinum, heart, great vessels, trachea, esophagus, vertebral body, or carina; or ipsilateral malignant pleural effusion NODES (N) N0 No regional lymph node metastases N1 Metastases to ipsilateral peribronchial or hilar nodes N2 Metastases to ipsilateral mediastinal or subcarinal nodes N3 Metastases to contralateral mediastinal or hilar, or to any scalene or supraclavicular nodes DISTANT METASTASES (M) M0 No distant metastases M1 Distant metastases The TNM subsets are subsequently grouped in a series of stages of disease to identify groups of patients with similar prognosis and therapy. STAGE Occult TX Stage I T1-2 Stage II T1-2 Stage IIIa Stage IIIb Stage IVAny T T N0 N0 N1 T3 T1-3 T4 T1-4 Any N N M0 M0 M0 N0-1 N2 N0-2 N3 M1 M M0 M0 M0 M0 36. After complete resection of Stage I non-small cell lung cancer (NSCLC), the role of adjuvant therapy is best summarized thus as: A. Postoperative radiation therapy improves disease-free survival. B. Postoperative radiation therapy improves overall survival. C. Postoperative chemotherapy improves disease-free survival. D. Postoperative chemotherapy improves overall survival. E. Adjuvant therapy is not indicated after complete resection of Stage I NSCLC. Answer: E DISCUSSION: Prospective randomized trials conducted by the Lung Cancer Study Group demonstrate that postoperative chemotherapy may be responsible for significantly longer disease-free survival in patients with Stage III (and perhaps Stage Asir Surgery MCQs Bank. © 1422H-2002- first impression © This project was raised after an idia by Dr. Gharama Al-Shehri (consultant surgeon). Developed and typed by Dr. Ghazi Al-Shumrani (intern). 107 Thoracic surgery II) NSCLC. The efficacy of postoperative chemotherapy and radiotherapy in patients with extensive lymph node involvement or positive surgical margins in reducing systemic recurrences and prolonging disease-free survival has also been demonstrated. Adjuvant therapy is not associated with improved overall survival and has not been shown to be beneficial in patients with Stage I NSCLC. Radiation therapy is an effective adjuvant treatment in many patients with carcinoma of the lung. Adjuvant radiotherapy, applied to patients with completely resected Stage II or Stage III (but not Stage I) NSCLC, has been shown to decrease local recurrence but has no significant effect on survival. However, postoperative irradiation may provide a survival advantage in patients who have resection and are found to have metastases to hilar or mediastinal lymph nodes. Thus, the purpose of adjuvant radiotherapy is prevention of local tumor recurrence, especially when lymph node sampling of the mediastinum at thoracotomy is incomplete. 37. Compared to segmentectomy or wedge resection, lobectomy for NSCLC is associated with: A. Similar operative morbidity but higher operative mortality. B. Similar operative mortality but higher operative morbidity. C. More severe postoperative pulmonary dysfunction. D. Lower incidence of locoregional recurrence. E. Equivalent locoregional recurrence. Answer: D DISCUSSION: The risk of recurrence after surgical resection according to the magnitude of the resection has been analyzed by the Lung Cancer Study Group. In a prospective, randomized trial involving more than 400 patients with T1N0 lung cancer, lobectomy was compared to segmentectomy and wedge resection. There was no significant difference in morbidity and mortality among the procedures. Furthermore, no difference was observed in postoperative pulmonary function between patients who underwent lobectomy and those who underwent lesser procedures. The rate of locoregional recurrence was significantly lower in patients who underwent lobectomy (5%) as compared with those who underwent either segmentectomy or wedge resection (15%). In another study, segmentectomy was compared to lobectomy in patients with Stage I lung cancer. In this study, the rate of locoregional recurrence was lower in patients who underwent lobectomy (5%), as compared with those who underwent segmentectomy (23%). Furthermore, there was a survival advantage in the patients undergoing lobectomy for T2 disease. 38. In contrast to NSCLC, small cell lung cancer (SCLC) is characterized by: A. Greater response rate to chemotherapy. B. Inability to achieve surgical cure. C. Less frequent association with paraneoplastic syndromes at the time of diagnosis. D. Lower likelihood of metastases present at the time of diagnosis. E. Slower growth. Answer: A DISCUSSION: For the purposes of staging, estimating prognosis, and selecting therapy, lung cancer is divided into two categories: NSCLC and SCLC. SCLC is characterized by more rapid growth, higher prevalence of metastases at the time of diagnosis, and greater responsiveness to chemotherapy and radiation therapy. After ascertaining the histological diagnosis of SCLC, staging is performed, including thorough neurological examination and CT evaluation of the chest, abdomen, and brain. For most patients with limited-stage disease, treatment is initiated with six cycles of combination chemotherapy. Radiotherapy to the chest is usually employed after three initial cycles of chemotherapy and is continued for 4 weeks. Among patients with limited-stage disease, thoracotomy for pulmonary resection is recommended for the subset of patients with stage I SCLC. 39. Which of the following statements about the diagnosis and staging of mesothelioma is/are correct? A. Fluid obtained by thoracentesis is usually adequate for accurate diagnosis. B. Open biopsy or thoracoscopy should be performed to obtain tissue for diagnosis. C. Immunohistochemistry should be performed in all cases of suspected mesothelioma. D. Chest CT and/or magnetic resonance imaging (MRI) are useful in the staging of mesothelioma. E. Head CT and bone scans are useful in the staging of mesothelioma. Answer: BCD Asir Surgery MCQs Bank. © 1422H-2002- first impression © This project was raised after an idia by Dr. Gharama Al-Shehri (consultant surgeon). Developed and typed by Dr. Ghazi Al-Shumrani (intern). 108 Thoracic surgery DISCUSSION: Approximately 90% of patients with mesothelioma develop pleural effusion, but cytologic specimens from pleural fluid are inaccurate for the diagnosis of mesothelioma, and open or thoracoscopic biopsy is required. Accurate diagnosis of mesothelioma is difficult: the epithelial variant must be differentiated from adenocarcinoma, whereas the sarcomatous form often resembles benign sarcomas. Immunohistochemistry using a panel of antibodies, and sometimes electron microscopy, is required for all cases. Relentless local spread is typical, and chest CT or MRI is essential to evaluate potential local extension into the chest wall, pericardium, mediastinum, or diaphragm. Metastatic disease is less common and occurs late (if at all) in the disease course, so head CT and bone scans are indicated only if clinical findings are suspicious for metastasis. 40. Which of the following statements about therapy for malignant pleural mesothelioma is/are correct? A. The role of surgery is confined to biopsy for diagnosis and pleurodesis for palliation of effusion. B. Extrapleural pneumonectomy involves resection en bloc of the lung, visceral and parietal pleura, pericardium, and diaphragm. C. If a lesion is unresectable by extrapleural pneumonectomy, pleurectomy/decortication is contraindicated. D. Neither surgery, chemotherapy, nor radiation therapy as a single therapy improves survival. E. Multimodality therapy, combining surgery, chemotherapy, and radiation therapy may improve survival in select patients. Answer: BDE DISCUSSION: In debilitated patients, palliation by pleurodesis is indicated; however, cytoreductive techniques, including pleurectomy/decortication, and extrapleural pneumonectomy, are indicated for patients who can tolerate surgery. For Stage I disease, extrapleural pneumonectomy is offered. If the patient cannot tolerate pulmonary resection or if the lesion is unresectable by extrapleural pneumonectomy, pleurectomy/decortication is appropriate. Both cytoreductive procedures, when used in a multimodality setting, may improve survival in selected patients. They also improve quality of life by relieving or delaying two severe symptoms of mesothelioma: dyspnea secondary to lung restriction by the tumor and pain from tumor invasion. No single modality (surgery, chemotherapy, or radiation therapy) improves survival. 41. All of the following may be acceptable operative approaches to management of the thoracic outlet syndrome except: A. Scalenectomy. B. Excision of a cervical rib. C. Thoracoplasty. D. First rib resection. E. Division of anomalous fibromuscular bands. Answer: C DISCUSSION: Supraclavicular decompression of the thoracic outlet is the preferred operative approach for the thoracic outlet syndrome. This procedure consists of extensive anterior scalenectomy, middle scalenectomy, removal of a cervical rib (if present), and, on occasion, first rib resection. Transaxillary first rib resection has been widely used as well but is associated with a greater risk for complications. Numerous fibromuscular anomalies have been described in association with the thoracic outlet syndrome. Thoracoplasty has no role in the management of this disorder. 42. Initial conservative (nonsurgical) management of the thoracic outlet syndrome may include all of the following except: A. Weight reduction. B. Improvement of posture. C. Exercises to strengthen the muscles of the shoulder girdle. D. Pentoxifylline. E. Avoiding hyperabduction. Answer: D DISCUSSION: The initial management of the thoracic outlet syndrome is nonoperative. A trial of weight reduction, shoulder girdle strengthening exercises, improvement of posture, and avoidance of hyperabduction should be recommended for 4 months or longer. These measures are successful in 50% to 70% of patients, particularly in young to middle-aged females with poor posture. Pentoxifylline is a hemorrheologic agent used in selected patients with peripheral arterial insufficiency and has no known benefit in the thoracic outlet syndrome. Asir Surgery MCQs Bank. © 1422H-2002- first impression © This project was raised after an idia by Dr. Gharama Al-Shehri (consultant surgeon). Developed and typed by Dr. Ghazi Al-Shumrani (intern). 109 Thoracic surgery 43. Which of the following statements about pectus excavatum are correct? A. It is the most common congenital malformation of the chest wall. B. The most frequent presenting complaint is the cosmetic deformity. C. The manubrium and first and second costal cartilages typically are involved in the deformity. D. It may be associated with cardiac defects and other skeletal defects such as scoliosis. E. Restrictive alterations in chest wall mechanics and abnormalities in pulmonary function tests have been documented. Answer: ABDE DISCUSSION: Congenital deformities of the chest wall represent a spectrum of deformities ranging from minor cosmetic defects to gross deformities incompatible with life. Pectus excavatum, or funnel chest, is the most common of the congenital deformities of the chest wall, accounting for 90% of such defects. It is characterized by a concave, posteriorly displaced sternum due to overgrowth of the costal cartilages. Most commonly the defect begins at the junction of the manubrium and the body of the sternum and becomes progressively deeper toward the xiphoid. The manubrium and the first and second costal cartilages typically are normal. The defects have both physiologic and psychologic consequences and are often associated with other abnormalities, including congenital heart disease, Marfan's syndrome, and other skeletal defects, including scoliosis. Patients most often present because of the cosmetic defect but frequently are found to have other symptoms, including impaired cardiopulmonary function and scoliosis. Pulmonary complaints include dyspnea and respiratory tract infections. Restrictive alterations in chest wall mechanics and abnormalities in pulmonary function tests, including decreased vital capacity, decreased total lung capacity, decreased maximal ventilatory volume, and decreased maximal breathing capacity, have been documented. 44. Surgical correction of pectus excavatum is characterized by which of the following? A. Significant cosmetic improvement initially but a high incidence of recurrence of the defect on late follow-up. B. An increase in exercise tolerance and respiratory reserve postoperatively. C. Improvement in FEV 1, vital capacity, and total lung capacity. D. Improvement in maximal ventilatory volume, total progressive exercise time, and maximal exercise capacity. E. Prevention of the development of “thoracogenic scoliosis.” Answer: BDE DISCUSSION: Because of the significant cosmetic and psychological improvement, subjective increase in exercise tolerance, documented improvement in cardiac and respiratory status, and prevention of the development of scoliosis following surgical intervention in these patients, surgical correction should be considered for all patients with moderate to severe deformity. Cosmetic results of surgical correction are excellent, and recurrence is uncommon. Objective improvement in cardiac function has been documented postoperatively, owing to relief of the sternal compression. Postoperatively, worsening of the FEV 1, vital capacity, and total lung capacity have been noted, whereas a significant improvement in maximal ventilatory volume, total progressive exercise time, and maximal oxygen consumption has also been documented. Following surgical correction there is a consistent increase in maximal exercise capacity at every level of workload, a lower heart rate at every workload, and an increase in exercise duration. 45. Which of the following statements about the diagnosis of chest wall tumors is/are correct? A. Pain is a common presenting symptom. B. Firmness and fixation to underlying bone and muscle are important to note in the physical examination as aids to diagnosis. C. In general, chest wall tumors are slow growing and produce symptoms late in their course. D. CT is the most useful imaging study for making the diagnosis and for planning surgical resection of chest wall tumors. E. Angiography should be performed routinely. Answer: BCD DISCUSSION: Seventy-five per cent of patients present with a slow-growing, painless chest wall mass. A firm mass that is fixed to an underlying rib is more likely to be of bony or cartilaginous origin. Conversely, soft, mobile tumors are more likely to be of soft tissue origin. CT defines depth of invasion and extent of tumor and is the most useful imaging modality. Angiography should be employed selectively, primarily for very large and vascular tumors. 46. Which of the following statements about chest wall resection and reconstruction is/are correct? Asir Surgery MCQs Bank. © 1422H-2002- first impression © This project was raised after an idia by Dr. Gharama Al-Shehri (consultant surgeon). Developed and typed by Dr. Ghazi Al-Shumrani (intern). 110 Thoracic surgery A. Most tumors of soft tissue and bone require 4-cm. margins to be adequately resected. B. At least one normal rib above and below the primary tumor should be included in the resection. C. Techniques of chest wall reconstruction are directed at the prevention of paradoxical chest wall movement with respiration. D. Soft tissue defects are most conveniently addressed by stretching the existing skin over the defect under tension. E. Chest wall defects that are covered by the scapula require no special reconstructive procedures, even if the defects are quite large. Answer: ABCE DISCUSSION: Margins of resection of chest wall tumors should be at least 3 cm. of skin, 4 cm. of muscle, and 6 cm. of bone. Old biopsy sites should be included in the specimen. A normal rib above and below the specimen should also be included. Prevention of paradoxical chest wall movement is the primary goal of chest wall reconstruction. Large soft tissue defects are best managed by myocutaneous pedicle flaps. In general, defects larger than 5 cm. require reconstruction. Defects covered by the scapula require no reconstruction. 47. Prolonged extracorporeal membrane oxygenation (ECMO): A. Is highly successful in the treatment of severe respiratory failure in newborn infants. B. Is contraindicated in adult respiratory distress syndrome (ARDS). C. Causes hemolysis and renal failure. D. Requires total systemic heparinization (activated clotting time longer than 500 seconds). E. Is identical to heart/lung bypass for cardiac surgery. Answer: A DISCUSSION: The survival rate of newborn infants who are moribund from respiratory failure with ECMO is 80% to 90%. ECMO is also indicated in ARDS with a survival rate from 40% to 50%. Hemolysis and renal failure are rare complications. ECMO requires low-dose partial heparinization, with clotting times in the range of 200 seconds. Several modifications in the conventional heart/lung machine permit the extension of ECMO from hours to days. 48. Indications for ECMO include: A. Newborn infants with pulmonary hypoplasia secondary to congenital diaphragmatic hernia. B. Meconium aspiration syndrome in full-term babies (at least 35 weeks). C. Children with pulmonary infection after bone marrow transplantation. D. Adults with acute viral pneumonia. E. Adults requiring mechanical ventilation and 100% oxygen for 2 weeks or longer. Answer: BD DISCUSSION: At present ECMO is not used for infants smaller than 1500 gm. because of a high risk of intracranial bleeding. ECMO is very successful in the treatment of respiratory failure in full-term newborn infants. Immunosuppression is a relative contraindication to ECMO. ECMO is indicated in adults with acute, potentially reversible respiratory failure, but mechanical ventilation and high oxygen concentration for more than 10 days are contraindications. 49. Venovenous ECMO: A. Avoids major arterial access. B. Provides cardiac and pulmonary support. C. Can be accomplished via cannulation at separate venous sites or at a single venous site using a double-lumen catheter. D. Provides greater venous drainage than venoarterial ECMO. E. Maintains the normal pulsatile blood flow to the systemic circulation. Answer: ACE DISCUSSION: Venovenous ECMO has become the access technique of choice for patients with respiratory failure without significant requirement for cardiac (hemodynamic) support. In neonates, a double-lumen cannula allows ECMO to be performed through a single incision over the right internal jugular vein. It can also be performed by separate cannulation of the femoral and jugular veins. In either configuration, venovenous ECMO avoids cannulation of any major arteries and maintains the normal pulsatile circulation through the heart and lungs. Venous drainage is no different with venovenous ECMO. Asir Surgery MCQs Bank. © 1422H-2002- first impression © This project was raised after an idia by Dr. Gharama Al-Shehri (consultant surgeon). Developed and typed by Dr. Ghazi Al-Shumrani (intern). 111 Thoracic surgery 50. As compared with venovenous ECMO, venoarterial ECMO: A. Requires cannulation of a major artery and vein. B. Provides both cardiac and respiratory support. C. Can be performed with less anticoagulation. D. Usually maintains a normal pulse pressure. Answer: AB DISCUSSION: Venoarterial ECMO can provide total cardiorespiratory support via cannulation of a major vein and artery (usually the right internal jugular vein and common carotid artery in neonates). With most roller and vortex pumps, the arterial inflow from the ECMO circuit is nonpulsatile, and therefore pulse pressure is often reduced or absent. Venoarterial ECMO requires the same degree of anticoagulation as venovenous techniques. 51. A 24-year-old male has new onset of chest pain. Chest films demonstrate a large anterosuperior mass. Appropriate evaluation should include: A. CT of the chest. B. Measurement of serum alpha-fetoprotein and beta–human chorionic gonadotropin. C. A barium swallow. D. A myelogram. Answer: AB DISCUSSION: Elevated levels of serum alpha-fetoprotein and beta–human chorionic gonadotropin are indicative of a malignant nonseminomatous germ cell tumor. Optimal therapy for such a tumor is based on a cis-platinum-containing chemotherapeutic regimen. After normalization of serum markers, resection of residual disease is performed. Extensive surgical procedures prior to chemotherapy are not warranted. Confirmation of the diagnosis can usually be obtained using needle biopsy techniques. In some institutions patients are treated based on elevated serum markers alone. CT imaging is useful to evaluate tumor invasiveness, airway compression, vascular involvement, and the likelihood of resectability. Barium swallow may be helpful in the evaluation of enteric cysts. Myelography may be useful in patients with posterior mediastinal masses to evaluate for spinal column involvement. 52. Systemic syndromes frequently associated with mediastinal tumors include: A. Myasthenia gravis. B. Hypercalcemia. C. Malignant hypertension. D. Carcinoid syndrome. Answer: ABC DISCUSSION: Myasthenia gravis occurs in 10% to 50% of patients with thymoma. The incidence with which myasthenia gravis occurs in patients with a thymoma increases with the age of the patient. In males over 50 and females over 60 years of age, the incidence appears to be greater than 80%. Hyperparathyroidism due to a mediastinal parathyroid adenoma is a cause of hypercalcemia. Although parathyroid glands may occur in the mediastinum in 10% of the patients, they are usually accessible through a cervical incision. A sternotomy is required infrequently, even in those patients with a mediastinal parathyroid gland. Most often the adenomas are found embedded in or near the superior pole of the thymus. Mediastinal paraganglioma may produce significant catecholamines, predominantly norepinephrine. Catecholamine production causes a classic group of symptoms associated with pheochromocytomas, including periodic sustained hypertension often accompanied by orthostatic hypotension, and hypermetabolism manifested by weight loss, hyperhydrosis, palpitation, and headaches. Mediastinal carcinoid tumors have been more frequently associated with Cushing's syndrome because of the production of adrenocorticotrophic hormones. These tumors uncommonly cause the carcinoid syndrome. 53. A 36-year-old female developed dyspnea on exertion that has progressed over 3 months. Chest film reveals a left anterior mediastinal mass with evidence of elevated left hemidiaphragm. CT indicates probable invasion of the pericardium. Paratracheal or subcarinal adenopathy is not identified. Appropriate intervention in this patient would include: A. A median sternotomy with radical resection of the tumor, sacrificing the left phrenic nerve and excising the involved pericardium. Asir Surgery MCQs Bank. © 1422H-2002- first impression © This project was raised after an idia by Dr. Gharama Al-Shehri (consultant surgeon). Developed and typed by Dr. Ghazi Al-Shumrani (intern). 112 Thoracic surgery B. A mediastinoscopy with biopsy. C. A left anterolateral thoracotomy or median sternotomy with generous biopsy of the tumor. D. Observation with repeat chest radiography in 3 months. Answer: C DISCUSSION: The differential diagnosis of an invasive anterosuperior mediastinal mass includes thymoma, lymphoma, germ cell tumor, undifferentiated carcinoma, and carcinoid tumors. These tumors often have a very similar histologic appearance, which may cause an inaccurate diagnosis based on light microscopy alone. Use of electron microscopy and immunohistochemistry may be necessary to correctly determine the specific histologic diagnosis. Frozen section should be used to determine adequacy of tissue biopsy. Histologic diagnosis based on frozen section examination in many of these tumors may be erroneous. Although radical resection of tumor is indicated for thymoma, chemotherapy and radiotherapy are the modalities used for the treatment of patients with lymphomas and germ cell tumors. Exact determination of tumor histology by permanent section should precede radical resectional therapy. Generous tissue biopsy is necessary for the precise subtyping of lymphomas. Mediastinoscopy is useful in patients with paratracheal and pericarinal masses or adenopathy, particularly when right-sided. Observation of a patient with invasive mediastinal mass is not warranted. 54. An 18-year-old male presents with a history of increasing shortness of breath that worsens in the recumbent position. On physical examination, the neck veins are noted to be distended, with facial plethora that is accentuated by lying the patient down. A 2.5-cm. left supraclavicular lymph node is palpable. Chest film reveals an extensive right anterosuperior mediastinal mass. Appropriate intervention may include: A. An urgent biopsy of the mediastinal mass under general anesthesia with subsequent initiation of therapy. B. CT. C. Pulmonary function testing in the sitting and supine positions. D. A biopsy of the right supraclavicular lymph node under general anesthesia. E. A biopsy of the supraclavicular lymph node under local anesthesia. Answer: BCE DISCUSSION: Although most patients with a mediastinal mass may undergo surgical procedures under general anesthesia with a minimal risk, patients with a large anterior, superior, or middle mediastinal mass, particularly those with posturerelated dyspnea and superior vena caval syndrome, have an increased risk of developing severe respiratory complications during general anesthesia. Useful techniques for identifying less symptomatic patients who have significant airway compression include CT imaging and pulmonary function tests. A reduction of the tracheal diameter by more than 35% on a CT scan and reduction of peak expiratory flow during pulmonary function testing are sensitive indicators of functional airway compression. In patients with airway compression and superior vena caval obstruction, the risk of general anesthesia is significant. Attempts to obtain a histologic diagnosis should be limited to needle biopsies or open procedures performed under local anesthesia. In situations in which histologic diagnosis cannot be obtained using these methods, therapy may be initiated with radiation, corticosteroids, and chemotherapy. However, a histologic diagnosis may not be obtainable in as many as 40% of these patients after initiation of treatment. Some proceed with biopsy of the mediastinal mass under general anesthesia. However, alterations in anesthetic management include: (1) induction of anesthesia in a semi-Fowler's or upright position, (2) availability of rigid bronchoscopy to allow reestablishment of an adequate airway, (3) use of a long endotracheal tube to allow advancement of the tube beyond the site of obstruction, (4) avoidance of muscle relaxants and the use of spontaneous ventilation when possible, (5) lower extremity intravenous cannulation, and (6) standby cardiopulmonary bypass. 55. A 42-year-old male who is scheduled to undergo elective knee surgery has a preoperative chest film that demonstrates a 5-cm. posterior mediastinal mass. The patient denies any neurologic symptoms and physical examination fails to elucidate any neurologic deficit. CT confirms the presence of a 5-cm. mediastinal mass in the left costovertebral gutter with minimal enlargement of the seventh thoracic foramen. Appropriate intervention includes: A. Resection of the posterior mediastinal mass using a standard posterolateral incision. B. A CT with myelography or magnetic resonance (MR) imaging. C. Two-stage removal of the tumor, performing the resection of the thoracic component first with subsequent removal of the spinal column component at a later date. D. One-stage removal of the dumb-bell tumor, excising the intraspinal component prior to resection of the thoracic component. Answer: BD Asir Surgery MCQs Bank. © 1422H-2002- first impression © This project was raised after an idia by Dr. Gharama Al-Shehri (consultant surgeon). Developed and typed by Dr. Ghazi Al-Shumrani (intern). 113 Thoracic surgery DISCUSSION: Approximately 10% of neurogenic tumors extend into the spinal column and are termed dumb-bell tumors because of the characteristic shape. Although 60% of patients with such tumors have neurologic symptoms related to spinal cord compression, the significant proportion of patients without symptoms underscores the importance of evaluating all patients with a posterior mediastinal mass for possible intraspinal extension. CT, MR imaging, and vertebral tomography may demonstrate an enlargement of the foramen, erosion of bone, or intervertebral widening, which are indicative of a dumb-bell tumor. If these findings are present, CT with myelography or MR imaging is indicated to evaluate the presence and extent of the intraspinal component. A one-stage removal of the tumor is recommended, with excision of the intraspinal component prior to resection of the thoracic component to minimize the risk of spinal column hematoma. 56. True statements regarding patients with a mediastinal mass include: A. Asymptomatic patients have a benign mass in over 75% of cases. B. Symptomatic patients are more likely to have a malignant lesion than a benign lesion. C. In a patient with a chest film demonstrating a mediastinal mass, a Tru-cut needle biopsy is a safe procedure. D. Seminomas usually produce alpha-fetoprotein. Answer: AB DISCUSSION: Seventy-six per cent of the asymptomatic patients with a mediastinal mass seen in one series over a recent 20-year period had a benign leison. In contrast, 62% of the symptomatic patients had a malignant neoplasm during this period. A number of intrathoracic and extrathoracic lesions may have an appearance similar to a primary mediastinal mass on routine chest films, as do a large number of cardiovascular lesions. Although angiography was used in the past for this differentiation, CT with contrast and MRI now distinguish a primary mediastinal mass from a cardiovascular lesion. Tru-cut needle biopsy of a cardiovascular lesion may be associated with significant hemorrhagic complications. Seminomas rarely produce beta–human chorionic gonadotropin and never produce alpha-fetoprotein. In contrast, over 90% of the nonseminomas secrete one or both of these hormones. 57. Which of the following would be the least appropriate in the management of acute suppurative mediastinitis? A. Wide débridement. B. Irrigation under pressure. C. Topical antibacterials. D. Long-term systemic antibacterials. E. Closure with muscle flaps. Answer: D DISCUSSION: Acute suppurative mediastinitis is a classic wound problem and forms a paradigm for principles of management. Wide débridement is perhaps the most important step in correcting this type of invasive wound sepsis. Drainage requires removal of tissue with vascular compromise. Tissue that is infected and can serve as an ongoing nidus for infection, particularly cartilage, must be removed. Irrigation is effective only when the irrigation fluid reaches into and flushes out débris and bacteria. The irrigation is insufficient if only dilutional and not also mechanically effective. Since infected tissue tends to become isolated from the systemic circulation the direct application of antibacterials reaches avascular areas. Some, such as silver sulfadiazine, penetrate avascular tissue better than, for instance, ointments or povidone iodine, and such an agent should be chosen. Wide débridement and the washing of debris with pressure irrigation make the wound then available to topical applications, which are often best packed into these deep, irregular cavities. Long-term systemic antibacterials serve no purpose and lead to potential resistant bacterial overgrowth. Although systemic antibacterials provide a measure of protection up to the margin where vascularized and nonvascularized tissues meet, topical agents are better in the actual infected site. Once closed, these wounds rapidly become sterilized. Even the infection at the bone level is far different from traditional osteomyelitis, and long-term systemic therapy is unnecessary. Muscle flaps are a great advance in closure technique, since they provide bulky protection, obliterate dead space, and help vascularize the wound. 58. Each of the following is appropriate for managing acute suppurative mediastinitis except: A. Alloplastic material and skin flaps. B. Rectus abdominis muscle flaps. C. Omentum. D. Pectoralis major muscle flaps. E. Rigid internal fixation. Asir Surgery MCQs Bank. © 1422H-2002- first impression © This project was raised after an idia by Dr. Gharama Al-Shehri (consultant surgeon). Developed and typed by Dr. Ghazi Al-Shumrani (intern). 114 Thoracic surgery Answer: A DISCUSSION: Alloplastic materials may be nonreactive in the laboratory and biologically acceptable in other areas (artificial hips, breast prostheses). Their introduction into a contaminated wound, however, would more likely promote rather than reduce infection. Various meshes and other types of “protection” devices are not necessary. Skin flaps alone do not obliterate dead space and have not been shown either to reduce or resist infection. The rectus abdominis muscle is a superb source of readily available tissue that can be rotated into very large cavities. The nature of the muscle allows it to be “dressed into” irregular cavities. It has an excellent, easily movable skin territory overlying it, which can also be transferred if locally available skin is wanting. The omentum has the great ability to fit into the many irregularities of some defects. For appropriately selected cases it is excellent. The pectoralis major muscle flaps are the usual initial choice since they are in the operative field. When the musculotendinous insertion is released their mobility is often sufficient. Additionally, it avoids the need for abdominal incisions. The latissimus dorsi muscle as a flap is dependable and includes sternal defects in the scope of its arc of rotation. It requires rotating the patient on the operating table and thus is less readily available than the other flaps. 59. Clinical features suggestive of myasthenia gravis include all of the following except: A. Proximal muscle weakness. B. Diplopia. C. Sensory deficits of the extremities. D. Dysphagia. Answer: C DISCUSSION: Weakness of proximal weight-bearing muscle groups is the hallmark of the clinical diagnosis of myasthenia gravis. The weakness or fatigue occurs with repetitive activity and improves with rest. The majority of patients (90%) experience ocular muscle involvement, manifested as diplopia or ptosis most easily demonstrated with sustained upward gaze. Cranial nerve involvement is uncommon but can be present, with symptoms of dysphagia, nasal regurgitation, and aspiration. Since myasthenia gravis is a disorder of neuromuscular transmission at the motor end plate, deep tendon reflexes and sensory examination are normal. 60. The diagnosis of myasthenia gravis can be confirmed most reliably using: A. Anti–acetylcholine receptor antibody titers. B. The Tensilon test. C. Electromyography (EMG). D. Single-fiber EMG. E. Physical examination. Answer: D DISCUSSION: Although findings from a careful history and physical examination are suggestive of the diagnosis of myasthenia gravis, specific diagnostic testing is required to confirm the diagnosis. Elevated anti–acetylcholine receptor antibodies are present in 85% to 90% of patients with generalized myasthenia but are often negative in patients with early or ocular myasthenia gravis. The Tensilon test is also positive in approximately 90% of patients with generalized myasthenia gravis, but both false-negative and false-positive results occur, especially in patients with mild or early disease. Standard EMG studies are helpful if positive, but their overall sensitivity may be as low as 35%. The specialized technique of singlefiber EMG is the most reliable diagnostic test, being abnormal in 90% of patients with mild disease and in virtually 100% in patients with severe generalized myasthenia gravis. 61. All of the following statements are true about the pathogenesis of myasthenia gravis except: A. The number of functional acetylcholine receptors at the motor end plate is reduced. B. An autoimmune mechanism involving antibodies to the acetylcholine receptor has been proposed. C. Complement system involvement has been demonstrated. D. A nonspecific “thymitis” may initiate the autoimmune response. E. Clinical improvement following thymectomy is correlated with decreased acetylcholine receptor antibody titers. Answer: E Asir Surgery MCQs Bank. © 1422H-2002- first impression © This project was raised after an idia by Dr. Gharama Al-Shehri (consultant surgeon). Developed and typed by Dr. Ghazi Al-Shumrani (intern). 115 Thoracic surgery DISCUSSION: Myasthenia gravis is generally regarded as an autoimmune disorder due to antibodies directed toward the acetylcholine receptor. A variety of autoimmune mechanisms have been proposed; the ultimate result is a reduction in the number of functional acetylcholine receptors at the motor end plate. Proposed immune mechanisms include complementmediated receptor destruction, antibody-induced accelerated receptor turnover, and simple receptor blockade. In spite of these proposed immune mechanisms, the severity of myasthenia symptoms and improvement following therapy do not correlate with antibody titers. Although the source of these autoantibodies is not proven, it is generally felt that a nonspecific thymitis may trigger the autoantibody response, the thymic myoid cells serving as the source of the antigen. 62. Which of the following statements about the relationship of the thymus and myasthenia gravis is/are true? A. Thymic abnormalities are present in up to 80% of patients with myasthenia gravis. B. Thymoma is present in up to 20% of patients with myasthenia gravis. C. Myasthenia gravis will occur in up to 60% of patients with thymomas. D. Myasthenia patients with thymoma respond more favorably to thymectomy. E. Thymoma is the most common abnormality of the thymus in patients with myasthenia gravis. Answer: ABC DISCUSSION: The central role of the thymus gland in the pathogenesis of myasthenia gravis is based on the observation that more than 80% of patients have histologic abnormalities of the thymus and on the beneficial effect of thymectomy on patients' symptoms. Of the patients with documented abnormalities of the thymus the majority have B-cell lymphoid hyperplasia; only 20% have a thymoma. Conversely, up to 60% of patients with known thymoma will have or ultimately develop myasthenia gravis. In these patients, with thymoma and myasthenia gravis, the response to thymectomy is less favorable than in those without thymoma. 63. Which of the following statements about the results of thymectomy for myasthenia gravis are true? A. Patients with ocular symptoms experience clinical improvement in 90% of cases. B. Clinical remission can be expected in 90% of cases. C. The response rate to thymectomy for patients with generalized symptoms is 90%. D. Patients with thymoma experience improvement in 75%. E. Continued medical therapy is required in 75%. Answer: C DISCUSSION: Overall, improvement can be expected in 90% of patients who undergo thymectomy for generalized myasthenia gravis. In general, the results are more favorable in patients with mild generalized myasthenia. In patients with only ocular symptoms, the benefit following thymectomy is less clear; improvement is documented in 80%. The response rate is even less (30%) in patients with thymoma. Complete remission occurs in 40% to 50% of patients following thymectomy, and the remainder require some continued medical therapy. 64. All of the following are true of the treatment of myasthenia gravis except: A. The transcervical approach to surgical thymectomy is less likely to benefit the patient with myasthenia gravis. B. Corticosteroids result in improvement in 80% of patients. C. Plasma exchange is associated with improvement in up to 90% of patients. D. Medical therapy with Mestinon (pyridostigmine) is associated with remission in approximately 10% of patients. E. Surgical thymectomy, regardless of the approach, is associated with improved remission and response rates as compared with medical therapy. Answer: A DISCUSSION: Although Mestinon therapy results in clinical improvement in most patients, complete remission can be expected in only 10%. In addition, intolerable side effects may limit their usefulness. In patients who fail to respond to Mestinon therapy, and in those who experience significant side effects, corticosteroids can be utilized, with improvement expected in 80% of patients. Plasma exchange results in improvement in 90% of patients, but the cost of therapy and its transient duration of benefit limit the use of pheresis therapy to special circumstances such as preoperative preparation or in myasthenic crisis. Overall, response rates to surgical thymectomy range from 80% to 95%, and complete remission occurs in 30% to 50%. This benefit following thymectomy has not been shown to depend on the particular technique utilized. Remission and response rates are similar for transcervical, standard transsternal, and the “maximal thymectomy” techniques. Asir Surgery MCQs Bank. © 1422H-2002- first impression © This project was raised after an idia by Dr. Gharama Al-Shehri (consultant surgeon). Developed and typed by Dr. Ghazi Al-Shumrani (intern). 116 Thoracic surgery 65. Which of the following is/are acceptable alternatives in the management of malignant pericardial effusion? A. Pericardiocentesis. B. Subxiphoid pericardiotomy (“pericardial window”). C. Thoracotomy with pericardiectomy. D. Instillation of tetracycline or bleomycin into the pericardial space. E. Treatment of the underlying malignancy. Answer: ABCDE DISCUSSION: In patients with symptomatic malignant pericardial effusions, management options may be designed to establish a diagnosis, relieve symptoms, or prevent recurrence. Pericardiocentesis is very successful in removing fluid for diagnosis and alleviating symptoms; however recurrence rates are greater than 50%. This rate can be reduced to around 20% with instillation of sclerosing agents such as tetracycline or bleomycin. Surgical techniques, including subxiphoid pericardiotomy and thoracotomy with pericardiectomy, offer the highest success rates (approximately 90%) but are more invasive and usually require general anesthesia. Systemic antitumor therapy with chemotherapy or radiation therapy can be effective in controlling malignant effusions in cases of sensitive tumors such as lymphomas, leukemias, and breast cancer. 66. Which of the following statements about cardiac tamponade is/are correct? A. At least 500 ml. of fluid must be present in the pericardium of an adult to cause symptoms of tamponade. B. A drop in systemic blood pressure of greater than 20 mm. Hg during inspiration (pulsus paradoxus) is a finding specific to cardiac tamponade. C. The vast majority of patients with cardiac tamponade demonstrate a low QRS voltage, nonspecific ST T-wave abnormalities, and electrical alternans (alternation of QRS amplitude) on the electrocardiogram. D. In trauma victims with cardiac tamponade, the three components of “Beck's triad” (hypotension, elevated jugular venous pressure (JVP), and muffled heart sounds) are almost always present. E. When the diagnosis is made, treatment must be instituted rapidly and may include pericardiocentesis, creation of a pericardial window, and identification and treatment of the underlying cause. Answer: E DISCUSSION: Development of tamponade symptoms depends on the rate of accumulation of fluid. As little as 100 to 200 ml. accumulating rapidly may cause symptoms, whereas a slowly developing pericardial effusion of over 1 liter may remain asymptomatic. Pulsus paradoxus is not specific for tamponade; it may occur in patients with severe congestive heart failure, chronic obstructive pulmonary disease, hypovolemia, acute pulmonary embolism, or shock. Electrocardiographic findings of low QRS voltage and nonspecific ST T-wave changes are common in this condition, but electrical alternans, often considered pathognomonic of cardiac tamponade, is present in only a small number of patients. Trauma victims with tamponade frequently lack one or more of the elements of Beck's triad; for example, associated hypovolemia may lead to low or normal jugular venous distention. Since cardiac tamponade is life threatening, therapy designed to drain the pericardial fluid must be provided quickly and the underlying cause must be established and controlled. 67. Which of the following statements about constrictive pericarditis is/are correct? A. Most patients who develop constrictive pericarditis after cardiac operation present with symptoms within 6 months of the procedure. B. Results of pericardiectomy for constrictive pericarditis are worse in patients who develop constriction after mediastinal irradiation. C. Drainage of asymptomatic pericardial effusions arising from acute pericarditis is advised to prevent development of constrictive pericarditis. D. If surgical treatment is planned for constrictive pericarditis it should involve total or complete pericardiectomy. E. Echocardiography can usually make the diagnosis by imaging a thickened pericardium. Answer: BD DISCUSSION: The time course in the development of constrictive pericarditis after cardiac surgery ranges from 1 month to nearly 9 years, but the mean interval from surgery to presentation is about 23 months. Most series have reported poorer outcomes from pericardiectomy for postirradiation constrictive pericarditis, possibly owing to underlying myocardial fibrosis. In this subset, 5-year survival averages 50%, as compared with 75% for constrictive pericarditis of all causes. Constrictive pericarditis is a rare complication of acute pericarditis. As a result, drainage of asymptomatic (nonpurulent) Asir Surgery MCQs Bank. © 1422H-2002- first impression © This project was raised after an idia by Dr. Gharama Al-Shehri (consultant surgeon). Developed and typed by Dr. Ghazi Al-Shumrani (intern). 117 Thoracic surgery pericardial effusions from acute pericarditis is not required. Patients with significant symptoms from constrictive pericarditis should undergo total pericardiectomy, even though this procedure carries an operative mortality rate of approximately 10%. Limited pericardiectomy has proven to be ineffective for this condition. It can be difficult to distinguish constrictive pericarditis from restrictive cardiomyopathy. Echocardiography may help by demonstrating chamber dimensions and wall motion abnormalities, but CT and MRI more accurately assess pericardial thickness. 68. The relationship between small-cell and non-small cell lung cancers can be described by the following: a. They differ by histology, clinical behavior and cell of origin b. Of all lung cancers, approximately 80% are non-small cell and 20% are small cell c. Both cell types are predictably responsive to chemotherapy d. The International Staging System can be applied to both tumor types e. The majority of non-small cell cancer patients vs. the minority of small cell cancer patients are candidates for pulmonary resection Answer: b Although small cell and non-small cell lung cancers do differ by histology and clinical behavior, they probably have a common origin since c-myc or n-myc amplified small cell lung cancer lines will undergo transition to non-small cell phenotypes after insertion of an activated ras/gene. The overall incidence of lung cancers is 80% non-small cell and 20% small cell. Only the small cell carcinoma is predictably responsive to chemotherapy. The staging system for small cell lung cancer is based on limited vs. extensive disease outside of a tolerable radiotherapy portal while the International Staging System uses TNM descriptors for 4 clinical stages. Unfortunately, only about 30% of patients with non-small cell lung cancer have potentially resectable tumors. 69. A 62-year-old male smoker presents with right anterior chest pain. There is a 3 cm mass attached to the chest wall with radiographic evidence of rib erosion and positive cytology for non-small cell carcinoma. Which of the follow is/are true: a. The patient is inoperable due to tumor size and chest wall involvement b. Radiation therapy is the preferred initial treatment c. Operative resection should be performed with en bloc removal of the tumor and adjacent chest wall as well as a mediastinal lymph node resection d. Positive mediastinal nodes will have little effect on survival e. The patient would be classified Stage IIIa Answer: c, e Survival after resection for non-small cell lung cancer is related to the stage of the disease with a strong adverse effect from nodal involvement. This is true even for large peripheral tumors that extend into the chest wall as in this case where a 40– 50% survival would be expected in the absence of nodes (T3N0:Stage IIIa) but only a 15% survival with nodal involvement. Radiation therapy would be a postoperative consideration to reduce the incidence of local recurrence. En bloc operative resection of the involved lobe and mediastinal nodes for staging would offer the greatest likelihood of cure. 70. For the patient in the pervious question to become an operative candidate which of the following must be met? a. Extrathoracic metastases must be able to be controlled by another modality, e.g. radiotherapy b. Tumor doubling time must exceed 40 days c. If there is recurrence at the primary site, it must be treated before the metastatic disease d. Even if effective systemic therapy is available, resection of metastases is preferred e. If pulmonary reserve is marginal, resection of the maximal number of metastatic foci should be performed Answer: c There are a number of controversial areas in the area of operative approaches to metastatic disease in the lung, but there is general agreement that any extrathoracic metastases preclude eligibility for pulmonary resection. Although tumor doubling time is a measure of its aggressiveness, it is too variable to have prognostic significance and is generally disregarded as a Asir Surgery MCQs Bank. © 1422H-2002- first impression © This project was raised after an idia by Dr. Gharama Al-Shehri (consultant surgeon). Developed and typed by Dr. Ghazi Al-Shumrani (intern). 118 Thoracic surgery criterion for resection. Primary site recurrence must be treated before the metastatic focus to prevent further seeding. If effective systemic therapy is available as would be expected in breast and testicular cancer or osteogenic sarcoma, it is preferred over surgical resection. Similarly, pulmonary resection should not be undertaken unless the pulmonary reserve will allow all metastatic foci to be resected. 71. Biopsy of the lesion in the previous question is reported as “bronchial carcinoid with no signs of atypia.” Which of the follow is/are true? a. Sleeve resection of the bronchus would be appropriate b. Lymph node biopsy at time of resection is unnecessary c. Associated carcinoid syndrome is very unlikely d. If carcinoid syndrome were found in a tumor this size, hepatic metastases would be likely e. When bronchial carcinoid syndrome occurs, right-sided cardiac valves are affected Answer: a, c, d In the absence of atypia, carcinoids are only locally malignant and can be managed by limited lung and/or bronchial resection. Therefore, a sleeve resection of the bronchus preserving distal lung would be appropriate. Lymph node sampling at the time of resection, however, is advisable to ensure that a complete resection has been performed. The carcinoid syndrome is rarely found except in the presence of a large primary tumor or hepatic metastases. When the carcinoid syndrome does occur, it is left-sided cardiac valves that are affected rather than right, which one would expect with gastrointestinalcarcinoids. 72. In the evaluation and preparation of a 55-year-old smoker for resection of a 3 cm pulmonary adenocarcinoma, the following is/are true: a. b. c. d. Preoperative cessation of smoking does not reduce postoperative pulmonary complications Resting PaCO2 is of more value than PaO2 FEV1 is of more value than measured vital capacity Diffusion capacity should be measured routinely e. V/Q lung scan is useful when pulmonary reserve is marginal Answer: b, c, e Preoperative cessation of smoking for a period of 2 weeks can reduce pulmonary complications and should be required. In the preoperative assessment for pulmonary resection, the PaCO2 is of more value than the PaO2 since an elevated PaCO2 > 50 mmHg identifies the very high risk patient with chronic lung disease. Hypoxemia may be secondary to the mechanical effects of the tumor producing ventilation/perfusion mismatch. The latter can be confirmed by V/Q lung scan which also serves to identify areas of functioning lung in patients with marginal pulmonary function. The best screening test for adequacy of pulmonary reserve is the FEV1. It identifies obstructive pulmonary disease which is more important than the restrictive lung disease identified by vital capacity measurement. Diffusion capacity measurement provides little additional information of value. 73. Following resection of a T1N1 squamous cell cancer in a 47-year-old male, the following is/are true: a. There is a higher risk of local recurrence than with any other histologic type of non-small cell cancer b. The greatest risk to the patient is a distant metastasis c. Of all metastatic sites, liver is most likely d. If the patient survives five years, there is a greater risk of a new lung cancer than recurrence e. To improve survival, the patient should be considered for adjuvant chemotherapy Answer: a, b, d The risk of local recurrence for non-small cell carcinomas of the lung is much more common for those of squamous cell histology than the others and averages 20%–30% overall. The greatest risk, however is of distant metastases which occur in 70%–80% of patients, regardless of stage. Almost all recurrences are seen within five years, and of the distant metastatic sites, the brain is most commonly affected. In this patient with Stage II disease, radiation therapy would be a consideration Asir Surgery MCQs Bank. © 1422H-2002- first impression © This project was raised after an idia by Dr. Gharama Al-Shehri (consultant surgeon). Developed and typed by Dr. Ghazi Al-Shumrani (intern). 119 Thoracic surgery to reduce the incidence of local recurrence, but not chemotherapy. After five years, the highest risk would be from a new lung cancer rather than a recurrence. 74. A 42-year-old woman with hemoptysis is seen to have a 2 cm mulberry appearing polypoid lesion in the left mainstem bronchus suspicious for bronchial adenoma. The differential diagnosis includes which of the following: a. Mucoepidermoid carcinoma b. Plasma cell granuloma c. Carcinoid tumor d. Adenoid cystic carcinoma e. Mucous gland adenoma Answer: all of the above The term bronchial adenoma includes a spectrum of tumors arising from epithelial stem cells which vary from the benign mucous gland adenoma to the malignant adenoid cystic and mucoepidermoid carcinomas as well as the carcinoid tumors of similar varied behavior. Among these variants, the carcinoid are most common representing 80%–90% of all bronchial adenomas. 75. A 42-year-old man has a solitary “coin lesion” 2 cm in diameter in the area of the right upper lobe on a routine chest radiograph. Which of the following is/are true? a. A previous radiograph from five years prior showing the lesion to be 1.2 cm in diameter indicates malignancy b. If a CT scan shows mediastinal adenopathy, mediastinoscopy is preferable to thoracotomy c. In the absence of previous radiographs, the lesion should be followed by serial films at 6 month intervals d. Calcification in a concentric or “popcorn” configuration denotes a benign lesion e. Needle aspiration showing “chronic inflammatory cells” denotes a benign lesion Answer: b, d In the evaluation of a solitary lung lesion, previous radiographs are important, particularly if the lesion is new. A coin lesion that is growing slowly does not necessarily indicate malignancy, since the most common benign tumor, hamartoma, has a variable pattern of slow growth and typically will show “popcorn” calcification. Concentric calcification is also most suggestive of a benign granuloma. In the absence of previous radiographs, the lesion must be assumed to be malignant until proved otherwise and should not be dismissed to follow-up. If a CT scan shows mediastinal adenopathy, then mediastinoscopy with biopsy is appropriate to make a diagnosis. Needle aspiration results of “chronic inflammatory cells” is non-diagnostic. 76. A 2 cm peripheral squamous cell carcinoma in the lung of a 60-year-old male with a pleural effusion positive for malignant cells would be classified as: a. T1N0M1 b. T3N0M0 c. T3N0M1 d. T4N0M0 e. T4N0M1 Answer: d, e The presence of a pleural effusion in association with a primary lung cancer is usually an ominous sign precluding surgical resection. However, if more than one sample of the effusion is negative for malignant cells and it is non-bloody, it can be considered unrelated to the tumor and excluded as a staging element. When the effusion cytology is positive, the tumor is considered T4 regardless of size or nodal status. 77. A 53-year-old woman who had a malignant tumor removed 2 years ago presents with a solitary lung nodule 1.5 cm in diameter. The following is/are true: Asir Surgery MCQs Bank. © 1422H-2002- first impression © This project was raised after an idia by Dr. Gharama Al-Shehri (consultant surgeon). Developed and typed by Dr. Ghazi Al-Shumrani (intern). 120 Thoracic surgery a. If the primary tumor originated in the breast, the lesion is most likely to represent a new primary lung cancer. b. If the primary tumor was melanoma, the lesion is most likely to be metastatic c. If the remainder of the lung fields are clear, a CT scan is unnecessary d. If the primary tumor was in the GI tract, there is very little chance that the lesion is a new primary lung cancer e. Fine needle aspiration should always be performed prior to resection of the lung lesion Answer: a, b A new pulmonary lesion in a patient with a history of a previously treated malignancy poses a diagnostic and therapeutic challenge. A CT scan should always be obtained since plain radiographs can detect lesions only 9 mm in diameter or greater. The lesion is most likely to be metastatic if the prior malignancy was sarcoma or melanoma and most likely to be a new primary lung cancer if the prior malignancy originated in the head, neck or breast. When the original lesion was in the GI or GU tract, there is an equal chance that it is metastatic or a new primary. Fine needle aspiration does not usually alter the plan for excision and is done only when the patient is not an operative candidate or desires to know the diagnosis. 78. A 61-year-old male presents with a painful mass 3.5 cm in diameter below the clavicle and attached to the chest wall. The following is/are true: a. A CT scan is the best study to determine rib destruction b. The lesion should be removed enbloc without biopsy to minimize the chances for local recurrence c. The chances are approximately 40% that the lesion is metastatic d. If it is metastatic, the most likely primary tumor is in the lung or pancreas e. Fortunately, less than 50% of chest wall tumors are malignant Answer: c Chest wall tumors are uncommon, accounting for only 1–2% of all body tumors. About 57% of chest wall tumors are primary, whereas 43% are metastatic. Solitary metastases most frequently arise from the thyroid gland, the GU tract and the colon. Overall, about 60% of chest wall tumors are malignant, most arising form bone or cartilage. The CT scan is of value in demonstrating the relationship between the mass and contiguous structures, but of little value in determining bone destruction because of the oblique course of the ribs. Specific rib films are most helpful. Now that multimodality therapy is available, core needle biopsies are recommended and have not increased the incidence of local recurrence. 79. Concerning the sternum, the following is/are true: a. The xiphoid process is the anterior border of the thoracic outlet b. Gladiolus is the body of the sternum c. The angle of Louis is at the level of the 2nd costal cartilage d. The 11th rib is attached via costal cartilage to the xiphoid e. The sterno-manubrial junction is at the level of T4 posteriorly Answer: a, b, c, e The sternum consists of 3 segments, the upper manubrium, the body or gladiolus, and the xiphoid process which ends in the rectus sheath and has no costal attachments. The xiphoid marks the anterior border of the thoracic outlet. The junction of the manibrium and body is the sternal angle or angle of Louis which corresponds to the level of T4 posteriorly and attaches to the 2nd costal cartilage anteriorly. 80. A 22-year-old woman recovering from a traumatic head injury is noted to have bright red bleeding when her tracheostomy is suctioned. The following is/are true statement(s): a. Antibiotics should be administered to treat the bronchitis b. Deflation of the tracheal tube cuff is a useful diagnostic maneuver c. If massive bleeding occurs, a finger should be used to compress the innominate artery against the sternum d. Operative treatment of a tracheoinnominate fistula includes resection and prosthetic replacement of the innominate artery e. Tracheal resection is usually required for a tracheoinnominate fistula to prevent recurrence Asir Surgery MCQs Bank. © 1422H-2002- first impression © This project was raised after an idia by Dr. Gharama Al-Shehri (consultant surgeon). Developed and typed by Dr. Ghazi Al-Shumrani (intern). 121 Thoracic surgery Answer: b, c The complication of tracheoinnominate artery fistula characteristically occurs in young women and is often heralded by bleeding during the tracheostomy suctioning. Deflation of the tracheal tube cuff confirms the diagnosis if massive bleeding occurs. At that point the tracheal tube cuff should be overinflated and a finger inserted into the tracheostomy incision to tamponade the bleeding. Throughout this, the airway must be protected. Operative repair through an upper sternal split requires resection of the innominate and coverage of the oversewn vessels with viable tissue since the wound is contaminated. No prosthetic material should be inserted and tracheal resection is not necessary. 81. A 52-year-old alcoholic with fever and a cough productive of purulent sputum is found to have the opacity on chest film as shown (Fig. 62-15). The following is/are true statement(s): a. The findings suggest a parapneumonic empyema b. If pus is found on aspiration of the pleural space, a chest tube should be placed c. If pus is found on aspiration, bronchoscopy is a necessary part of the patient’s evaluation d. In this situation, rib resection for drainage is preferred to a large-bore chest tube e. Decortication of the lung should be considered if the lung fails to expand within 4 weeks Answer: a, b, c The posterior location of the infiltrate and fluid collection is typical of a parapneumonic empyema. The most important test is pleural aspiration which will usually yield frank pus, at which time a chest tube should be placed. Formerly, oily Dionosil was used to perform an empyemagram; this substance is now no longer commercially available. In the case of parapneumonic empyemas, tube drainage alone may be sufficient to allow full expansion of the lung. If this is not the case, a formal rib resection or early decortication should be performed. Decortication or marsupialization is indicated if the lungs fail to expand after 6–8 weeks. Every patient with spontaneous empyema should undergo bronchoscopy to rule out endobronchial obstruction by foreign body or tumor. 82. The lesion shown (Fig. 62-6) was found on a 32-year-old male on a routine chest film required for his employment. Which of the following is/are true? a. The stippled calcification and intact cortex of the rib are characteristic of osteochondroma b. The stippled calcification is characteristic of osteogenic sarcoma c. If the lesion is osteogenic sarcoma, the optimal treatment is resection and radiation therapy d. If the lesion is an osteochondroma, it need not be resected in this age group e. The radiographic picture is typical for Ewing sarcoma Answer: a Osteochondroma is the most common benign rib tumor and has a 3:1 male incidence. The stippled calcification and intact rib cortex are characteristic for this lesion in contrast to the bone destruction of Ewing sarcoma and combined bone destruction and “sunburst” calcification of osteogenic sarcoma. For both Ewing and osteogenic sarcoma, multimodality therapy using preoperative chemotherapy followed by resection yields better results than with radiation therapy. Osteochondromas in prepubertal children can be observed unless they become painful or enlarged, but are routinely resected in adults. 83. To resect a chondrosarcoma of the chest wall in a 42-year-old man, ribs 2–4 were removed, leaving a defect 8 x 8 cm. For reconstruction, the following is/are true: a. If this were to be posterior, beneath the scapula, reconstruction would not be required b. If this defect is anterior, the primary benefit of reconstruction is an improved cosmetic result c. Whenever chest wall reconstruction is considered, it should be delayed 6–12 months to allow detection of recurrent tumor d. If Marlex is used for reconstruction, no wound drainage tube is necessary e. If PTFE is used for reconstruction, both pleural and wound tubes should be used Answer: a, d, e Asir Surgery MCQs Bank. © 1422H-2002- first impression © This project was raised after an idia by Dr. Gharama Al-Shehri (consultant surgeon). Developed and typed by Dr. Ghazi Al-Shumrani (intern). 122 Thoracic surgery Skeletal chest wall defects that are full-thickness and occur posteriorly where they can be covered by the scapula do not require reconstruction. Anterior chest wall defects do require reconstruction, primarily to stabilize the chest wall and prevent paradoxical motion. The reconstruction should be immediate for optimal physiological benefit. Since Marlex mesh is porous, only a wound catheter is needed as pleural fluid will drain through it. PTFE, however, is a solid sheet necessitating both pleural and wound drainage. 84. An upright chest film of a cachectic, homeless 47-year-old woman shows blunting of the right costophrenic angle. The following is/are true: a. A lateral decubitus film should be obtained to confirm the presence of fluid rather than a CT scan b. Tuberculous effusion can readily be identified by stain and culture of aspirated fluid c. A pleural fluid glucose level lower than in the serum is diagnostic of empyema d. Bloody pleural effusion in this patient is diagnostic of an underlying malignancy e. Pleural fluid cytology report of lumphoma should be viewed with skepticism Answer: a, e Although the CT scan is a very sensitive indicator of pleural effusion, a lateral decubitus is the simplest way to differentiate fluid from pleural thickening or fibrosis. Tuberculous pleuritis is difficult to diagnose by stain or culture which have a 30% yield, but the diagnosis is facilitated by needle biopsy of the pleura. Pleural fluid glucose lower than in serum is characteristic of rheumatoid arthritis, neoplasms, and tuberculosis as well as empyema. A red-tinged fluid can occur from needle trauma, but even frankly bloody fluid in this patient may reflect trauma as well as underlying malignancy. Pleural inflammation induces reactive changes in mesothelial cells that makes them resemble lymphocytes, so a lymphoma diagnosis is suspect. Following shotgun wound of the chest wall, a 39-year-old woman desires reconstruction without a foreign-body prosthesis. Old incisions prohibit use of her rectus abdominus muscles. Considering chest wall muscles for reconstruction, the following is/are true statement(s): 85. The pectoralis major muscle is available and innervated by the medial and lateral pectoral nerves so named because it describes their relationship to the pectoralis minor a. The serratus anterior muscle is available since its absence has no functional significance b. There is no serratus posterior muscle c. The latissimus dorsi muscle is available and supplied by the thoracodorsal artery d. The latissimus dorsi is innervated by the thoracodorsal nerve with fibers from C6, C7 and C8 Answer: d, e The pectoralis major muscle can be used for reconstruction but the medial and lateral pectoral nerves are named from their respective cords of the brachial plexus. The serratus anterior muscle holds the scapula to the chest wall and its absence produces the functional and cosmetically disabling winged scapula. The serratus posterior muscle is attached to the 7th cervical and first three thoracic vertebrae posteriorly and functions as an accessory muscle of respiration. The constancy of the vascular pedicle to the latissimus dorsi and its size allow this muscle to be used to reconstruct defects of the head, neck, chest wall and pleural cavity. It is innervated by the thoracodorsal nerve with fibers from C6, C7 and C8. 86. A 38-year-old man presents with facial and upper extremity edema, venous distention in the neck and arms and a cyanotic appearance. The following is/are true statement(s): a. The most likely cause of the problem is mediastinal granulomatous disease b. A venogram should be obtained to confirm the diagnosis c. Mediastinoscopy for diagnosis is contraindicated d. If a malignancy is identified, resection is indicated for palliation e. If the etiology is benign disease, gradual improvement without operation is to be expected Answer: e Asir Surgery MCQs Bank. © 1422H-2002- first impression © This project was raised after an idia by Dr. Gharama Al-Shehri (consultant surgeon). Developed and typed by Dr. Ghazi Al-Shumrani (intern). 123 Thoracic surgery Although mediastinal granulomatous disease is one cause of the superior vena cava syndrome described, the most common cause (75%) is malignant disease. A venogram adds little information to the typical findings and increases risk from extravasation of contrast medium subcutaneously from the venous hypertension. Mediastinoscopy can be used for diagnosis with recognition of increased risk of bleeding and airway problems from the edema associated with the endotracheal intubation required for the procedure. If a malignancy is found, operative resection is usually precluded by the extent of mediastinal invasion. Fortunately, in the case of benign disease, the symptoms tend to improve with time as chest wall and mediastinal collaterals enlarge. 87. A 39-year-old woman with hypertension and radicular chest wall pain was found to have the lesion seen on chest radiograph (Fig. 63-23). The following is/are true statement(s): a. The location of the lesion suggests a teratoma b. High urinary vanillylmandelic acid levels would indicate that the lesion is a paraganglioma c. If the lesion was seen on a film 5 years earlier, resection would not be indicated d. A neurosurgical consultation should be obtained e. Vasoactive intestinal polypeptide level elevation suggests a ganglioneuroma Answer: d, e The posterior mediastinal location of the tumor is most indicative of a neurogenic tumor while teratomas are characteristically found in the anterior mediastinum. Neurogenic tumors can undergo malignant degeneration and should be resected, particularly in this symptomatic patient even if known to be present for years. The radicular pain suggests the possibility of intraspinous extension of the tumor, and therefore a neurosurgical consultation is appropriate. Both urinary vanillylmandelic acid elevation and vasoactive intestinal polypeptide can be produced by ganglioneuroma but would not be characteristic of a paraganglioma. Asir Surgery MCQs Bank. © 1422H-2002- first impression © This project was raised after an idia by Dr. Gharama Al-Shehri (consultant surgeon). Developed and typed by Dr. Ghazi Al-Shumrani (intern). 124